LSAT and Law School Admissions Forum

Get expert LSAT preparation and law school admissions advice from PowerScore Test Preparation.

 mikewazowski
  • Posts: 10
  • Joined: Oct 20, 2020
|
#80770
Hi,

I had a lot of trouble with this one. I easily crossed off (E) because it seemed to be identical to the first sentence of the passage (which is the conclusion?). I thought, 'we need an assumption and this isn't an assumption.' The first sentence says that the wages of many of the lowest-level employees would be protected from cuts with the max wage law. (E) states that if the law was enacted, one or more executives would not cut the pay.

First sentence: max wage law -> many protection from cuts
(E): max wage law -> one or more not cut

Much of you will be protected from rain with an umbrella = if you use an umbrella, some of you will not be rained on

Please let me know where I went wrong.
 Adam Tyson
PowerScore Staff
  • PowerScore Staff
  • Posts: 5153
  • Joined: Apr 14, 2011
|
#81199
It sometimes feels like an Assumption answer or a Must Be True answer is just repeating information from the stimulus, Mike, when in reality those answers are actually just providing links between two ideas that the author didn't completely link. It can help in cases like this to simplify the argument with a paraphrase. In this argument, the premise is the last sentence, and could be paraphrased as "this kind of law will give executives a reason to maintain worker pay." The conclusion in the first sentence can be paraphrased as "such a law will protect worker pay." What's the missing link? That executives will act based on that incentive! It may seem obvious, but the author never actually connected the two ideas. Instead of stating that executive would behave in this way, the author merely assumed that they would.

To use your analogy, this would be like an argument that said "Going out in the rain with an umbrella will keep you from getting rained on, because an open umbrella over your head keeps the rain off of you." The assumption is that if you go out in the rain with an umbrella you will hold it open over your head! Seems obvious, but it was never clearly stated.
User avatar
 JPConstantine
  • Posts: 6
  • Joined: Jun 30, 2022
|
#102533
Is answer A a shell game answer? It seems like, as stated before, one assumption is that CEOs make 50 times more than there lowest paid employees. If this is not true they could still cut pay for employees and raise their salaries within that ball park. That is why I am reviewing question A and thinking that at the very end is a shell game with the wording?
 Rachael Wilkenfeld
PowerScore Staff
  • PowerScore Staff
  • Posts: 1358
  • Joined: Dec 15, 2011
|
#102620
That word "all" in answer choice (A) poses a problem for you, JP. Remember that correct assumption answer choices are required for the argument. It's hard to prove language like "all" or "none." If the answer choice says "all" you negate it by stating changing the all to a not all. Here's the thing with this argument: we don't need it to be true for ALL of the lowest-paid employees to be working where executives make more than 50X the salary of the lowest-paid worker. Our stimulus only says that "many" would be protected. That doesn't require that "all" be protected. Therefore, answer choice (A) is not required. It doesn't matter if not all workers would fall into the category in the stimulus. We just need to know that many would.

Hope that helps!
User avatar
 sqmusgrave
  • Posts: 16
  • Joined: Sep 16, 2023
|
#103775
Can someone please explain to me how AC E isn't a restatement of the authors argument? I chose E by elimination but struggled with it because it just seemed like exactly what the argument was saying, and I thought we were supposed to choose AC's that are restatements of Premises or Conclusions. It's just saying "if we enact this law then the lowest paid corporate employees won't have their wages cut". This is what the argument is saying. I see in the response that an assumption is that they won't break the law, but I wouldn't have ever thought of this as an assumption! To me this kind of assumption is the type of baby-assumption that is prevalent in LR Q's, but these are ones that are so obvious, or so much of a given, that we don't really have to take them into account. Like if there was an argument that Child labor laws will prevent children from working overtime by making it illegal to do so, then it seems like a stretch to have to think "ah but what if no one obeys this law, so child labor laws must not really work".
Thanks!
 Luke Haqq
PowerScore Staff
  • PowerScore Staff
  • Posts: 722
  • Joined: Apr 26, 2012
|
#103818
Hi sqmusgrave!

This stimulus suggests that a maximum wage law would remove an incentive for executives to cut the wages of their lowest-paid employees, and thus reasons that the wages of the lowest paid corporate employees will be protected from cuts if such a law were enacted.

However, there's nothing in the stimulus that directly connecting the law to what executives choose to do. For example, it's possible that, even if a maximum wage law were enacted in the given country, some executives might still choose to cut the pay and benefits of the lowest paid corporate employees. If this were true, the argument that the lowest paid would be protected from cuts would fall apart.

That explanation is effectively applying the Assumption Negation technique to answer choice (E). Negated, the answer choice would be, "If such a maximum wage law were enacted in the economist’s country, one or more corporate executives would ... cut the pay and benefits of their corporations’ lowest-paid employees in the economist’s country."

Get the most out of your LSAT Prep Plus subscription.

Analyze and track your performance with our Testing and Analytics Package.